30
Chương 1: Các vấn đề về Bất Đẳng Thức AM-GM : I. Bất Đẳng Thức AM-GM Trong tiết này, chúng ta sẽ giới thiệu BĐT AM-GM mà các bạn học sinh phổ thông quen gọi với cái tên gọi đó là Bất Đẳng Thức Cô si . Trước hết ta xét trong những trường hợp đơn giản nhất . Đầu tiên, ta bắt đầu từ hằng đẳng thức 2 0 (a b) .Điều này tương đương với 2 2 2 a b ab .Dấu đẳng thức xảy ra khi và chỉ khi a b . Ta phát biểu lại theo ngôn ngữ toán học của BĐT AM-GM ( Trung bình cộng và trung bình nhân ): Cho hai số thực dương a,b . Lúc này ta có BĐT sau : 2 a b ab . Dấu đẳng thức xảy ra khi và chỉ khi a b . Nói một cách nôm na là Trung bình cộng luôn luôn lớn hơn trung bình nhân . Bỏ qua hình thức rất đơn giản nhưng BĐT AM-GM lại có những ứng dụng rất rộng rãi trong việc chứng minh các bài toán về cực trị và cả trong các bài toán cực trị hình học . Chúng ta hãy thử xét qua các ví dụ nhập môn sau : Ví dụ 1: Cho 0 a,b,c .CMR : 3 3 a b c abc Lời giải Đây chính là BĐT AM-GM cho 3 số dương . Lời giải của nó củng sẽ dựa trên cách áp dụng AM-GM với 2 số . Bất đẳng thức đã cho tương đương với : 3 3 4 P a b c abc abc Ta có : 4 3 3 3 2 2 4 4 P ab c abc abc abc abc (đpcm) Chúng ta thử một cách tiếp cách khác để chứng minh VD này . THCS ta đã biết được đẳng thức quan trọng sau : 3 3 3 2 2 2 1 3 0 2 a b c abc (a b c)((a b) (b c) (c a) )

Chương 1: Các vấn đề về Bất Đẳng Thức AM-GMk2pi.net.vn/data/files3/K2PI---SIeU PHaM BaT daNG THuC...1 2 1 2 b c c b c a a c Cộng lại, ta có điều phải chứng

Embed Size (px)

Citation preview

Chương 1: Các vấn đề về Bất Đẳng Thức AM-GM :

I. Bất Đẳng Thức AM-GM

Trong tiết này, chúng ta sẽ giới thiệu BĐT AM-GM mà các bạn học sinh phổ thông quen gọi

với cái tên gọi đó là Bất Đẳng Thức Cô si .

Trước hết ta xét trong những trường hợp đơn giản nhất .

Đầu tiên, ta bắt đầu từ hằng đẳng thức 2 0(a b) .Điều này tương đương với

2 2 2a b ab .Dấu đẳng thức xảy ra khi và chỉ khi a b .

Ta phát biểu lại theo ngôn ngữ toán học của BĐT AM-GM ( Trung bình cộng và trung bình

nhân ):

Cho hai số thực dương a, b . Lúc này ta có BĐT sau :

2

a bab

.

Dấu đẳng thức xảy ra khi và chỉ khi a b .

Nói một cách nôm na là Trung bình cộng luôn luôn lớn hơn trung bình nhân . Bỏ qua hình

thức rất đơn giản nhưng BĐT AM-GM lại có những ứng dụng rất rộng rãi trong việc chứng

minh các bài toán về cực trị và cả trong các bài toán cực trị hình học .

Chúng ta hãy thử xét qua các ví dụ nhập môn sau :

Ví dụ 1: Cho 0a,b,c .CMR :

3

3

a b cabc

Lời giải

Đây chính là BĐT AM-GM cho 3 số dương . Lời giải của nó củng sẽ dựa trên cách áp dụng

AM-GM với 2 số .

Bất đẳng thức đã cho tương đương với :

3 34P a b c abc abc

Ta có :

43 3 32 2 4 4P ab c abc abc abc abc (đpcm)

Chúng ta thử một cách tiếp cách khác để chứng minh VD này .

Ở THCS ta đã biết được đẳng thức quan trọng sau :

3 3 3 2 2 213 0

2a b c abc (a b c)((a b) (b c) (c a) )

Như vậy ta cũng suy ra được điều cần chứng minh .

Dường như ngay từ đầu tiên, người ta đã xây dựng các Bất Đẳng Thức dựa trên điều hiển

nhiên sau 2 0x .Các bạn hãy đọc hết cuốn sách này để tự trả lời câu hỏi này nhé !

Một số dạng tương tự của BĐT AM-GM cho 3 số :

Với 0a,b, c . Ta có :

3

3 3 3

13

23

a b cabc ( )

a b cabc

Sau đây chúng ta sẽ chứng minh Bất Đẳng Thức AM-GM trong trường hợp tổng quát nhất.

Có khoảng hơn 20 cách chứng minh cho BĐT AM-GM trong trường hợp tổng quát . Mà

chúng tôi không thể nào trình bày hết trong cuốn sách này được dù rất muốn trình bày . Đa

phần chúng ta đã bỏ qua việc xây dựng lý thuyết khi học toán mà tập trung vào các kĩ năng

tính toán . Điều này sẽ giúp các bạn đi nhanh lúc đầu nhưng sẽ hạn chế khả năng tuy duy và

giải toán dài lâu sau này của các bạn .

Với những Bất Đẳng Thức nhiều biến số, thì tư tưởng cơ bản và tự nhiên nhất đó chính là sử

dụng phép quy nạp. Tại sao lại thế ? Bởi vì luôn có một sự liên hệ giữa trường hợp 1n và

n . Như vậy nếu như chúng ta chứng minh được với trường hợp số liền trước thì với

trường hợp số liền sau ta hoàn toàn có cơ sở chúng minh được . Phương pháp quy nạp

chính là “chìa khóa vàng” của các nhà toán học khi xây dựng nên các lý thuyết toán học cổ

điển và hiện đại . Song, điều chúng ta cần nắm đó là cách quy nạp như thế nào . Đó là cả

một nghệ thuật . Chúng ta hãy theo giỏi lời giải sau của TS.Trần Nam Dũng ( ĐHKHTN-

ĐHQG Tp HCM ):

(Chứng minh bất đẳng thức Cauchy bằng quy nạp tiến).

ĐẶT MUA SÁCH

Link đăng ký: http://goo.gl/forms/5SbEpf57U9

Mua trực tiếp liên hệ Nguyễn Văn Quốc Tuấn số điện thoại: 0989631669

Facebook: https://www.facebook.com/chicanemhanhphucS2g

Ai có nhu cầu sẽ được chính tác giả ký tặng nhé!!!!!!!!!

Đặt mua sách Bí quyết tiếp cận hiệu quả Kỳ thi THPT quốc gia Bất đẳng thức- Giá trị lớn nhất

nhỏ nhất.

Nếu mua qua đây sẽ được tác giả ký tặng!!!

------GIÁ SÁCH---

Lưu ý: Mua nhóm trên 5 cuốn được miễn phí cước Chuyển phát nhanh.

Bí quyết tiếp cận hiệu quả Kỳ thi THPT quốc gia Bất đẳng thức- Giá trị lớn nhất nhỏ nhất. Giá

bìa 234.000đ

----THANH TOÁN----

TỔNG TIỀN THANH TOÁN = 200K +40K phí Chuyển phát nhanh, Bạn nào lựa chọn thanh

toán COD Qua bưu điện thì mất thêm 15K tiền thu hộ cho nhân viên bưu điện và cần thanh

toán trước 20K bằng cách gửi MÃ THẺ CÀO +SERI thẻ cào điện thoại vào số 0989631669.

---CÁCH THỨC THANH TOÁN ---

Hình thức 1: CHUYỂN KHOẢN

Thông tin tài khoản của thầy:

Các bạn chuyển vào tài khoản sau:

+Chủ thẻ: Nguyễn Văn Quốc Tuấn

+Ngân hàng TMCP Công Thương Việt Nam (Viettinbank) - Loại tài khoản: A - TK ATM

Số TK: 711AB2793863

HÌNH THỨC 2: THANH TOÁN BẰNG THẺ CÀO:

Sau khi đặt sách các bạn Gửi Mã thẻ cào + Số Seri (áp dụng với tất cả các loại thẻ của nhà

mạng) vào số điện thoại 0989631669. Lưu ý: Thanh toán bằng thẻ cào các em thanh toán 130%

tổng giá thanh toán gồm 130%*(200 ngàn +40 ngàn phí ship)

Lưu ý: Mua nhóm trên 5 cuốn được miễn phí vẫn chuyển

Ví dụ 2: Cho 1 2 na , a , , a là các số thực không âm. Chứng minh rằng ta luôn có

1 2 1 2n

n na a ... a n a a ...a

Lời giải

Trong các tài liệu, bất đẳng thức này thường được chứng minh bằng phép quy nạp lùi, hay quy

nạp kiểu Cauchy. Ở đây chúng ta trình bày một phép chứng minh khác.

Cơ sở quy nạp với n = 1, 2 được kiểm tra dễ dàng. Giả sử bất đẳng thức đã được chứng minh

cho n số. Xét n+1 số không âm a1, a2, …, an+1. Đặt a1a2…an+1 = An+1. Nếu tất cả các số bằng nhau

thì bất đẳng thức đúng. Trong trường hợp ngược lại, phải tồn tại hai số ai, aj sao cho ai < A <

aj. Không mất tính tổng quát, có thể giả sử an < A < an+1. Khi đó ta có (an – A)(an+1 – A) < 0, suy

ra an + an+1 > anan+1/A + A. Từ đó ta có

a1 + a2 + …+ an + an+1 > a1 + … + an-1 + anan+1/A + A (1)

Bây giờ áp dụng bất đẳng thức Cauchy cho n số a1 + … + an-1 + anan+1/A ta được

11 2 1 1 2 1

n nnn n n

a aa a ... a a n a a ...a nA

A

Kết hợp với (1) ta được đpcm.

Ví dụ 3: Với 0a,b,c thõa mãn điều kiện : 1a b c

b c a .Chứng minh rằng

1b c a

a b c

Lời giải

Ta có

1 2a b c b

b c a a

Tương tự:

1 2

1 2

b c

c b

c a

a c

Cộng lại, ta có điều phải chứng minh .

Bình luận: Lời giải là sự kết nối giữa giả thiết và điều phải chứng minh . Để ý quan sát ta thấy

nếu như cứ nhân 2 số hạng ở biểu thức điều kiện rồi lấy căn thì ta được một số hạng ở biểu

thức cần chứng minh Chính điều này là xuất phát điểm của lời giải như trên .

Ví dụ 4: Cho . Chứng minh rằng :

2 1 1 1a b c( ) (a b c)( )b c a a b c

Lời giải

Bất đẳng thức trên tương đương với:

2 2 2

2 2 23

a b c a b c c a b

b c a c a b a b c

Trước hết ta có : 3a b c

c a b theo AM-GM .

Bây giờ , ta xử lý tiếp một “đoạn đường” còn lại nữa .

Ta thử xem liên hệ của 2

2

a

b và

a

b là gì ?

Ta mạnh dạn áp dụng AM-GM thử xem :

2

2

2

2

2

2

1 2

1 2

1 2

a a

b b

b b

c c

c c

a a

Để ý rằng : 3a

b

Cộng lại ta được điều phải chứng minh.

Bình luận: Việc chèn thêm tham số trong việc áp dụng BĐT AM-GM là kĩ năng quan trọng

mà các bạn cần phải có trong việc giải toán Bất Đẳng Thức . Nhưng lưu ý khi chèn thêm tham

số các bạn phải đảm bảo được việc dấu đẳng thức xảy ra .

, , 0a b c

Ví dụ 5: Cho các số thực dương a, b,c thõa mãn : 3ab bc ca .Chứng minh rằng :

6 6 6 6 6 61 1 1 3 3a b b c c a

Lời giải

Trước hết dự đoán điểm rơi của bài toán là 1a b c .

Như vậy ta thử liên kết điều cần chứng minh và biểu thức điều kiện bằng cách áp dụng BĐT

AM-GM

6 6 2 21 3a b a b

Suy ra: 6 6 1 3a b ab

Tương tự:

6 6

6 6

1 3

1 3

b c bc

c a ca

Cộng tất cả các BĐT lại ta có đpcm .

Dấu đẳng thức xảy ra tại 1a b c

Bây giờ, chúng ta sẽ chuyển tiếp các Ví Dụ mà qua đó chúng ta sẽ thực hành được các phép

biến đổi cơ bản trong việc Áp dụng BĐT AM-GM .

Ví dụ 6: Cho a, b,c là các số thực dương .Chứng minh rằng :

4 4 4a b cabc

a b c

Lời giải

Ở bài toán này chúng ta sẽ sử dụng cách nhóm đối xứng để hạ bậc BĐT AM-GM .

Cụ thể, ta có :

4 4 4 4 4 44 4 4 2 2 2 2 2 2

2 2 2

a b b c c aa b c a b b c c a

Áp dụng tương tự, ta củng sẽ có :

2 2 2 2 2 2 2 2 2 2 2 22 2 2 2 2 2

2 2 2

a b b c b c c a c a a ba b b c c a abc(a b c)

Như vậy ta suy ra được đpcm . Dấu đẳng thức xảy ra khi và chi khi a b c

Ví dụ 7: Với a, b,c là các số thực không âm, chứng minh rằng :

21

3a bc b ca c ab (a b c)

Lời giải

Củng bằng cách nhóm đối xứng như ở VD6 , ta có :

2 2 2 2

2 2 2

2 2 2 3

ab bc bc ca ca ab(a b c) (a bc) (b ca) (c ab)

a bc b ca c ab b ac c ab a ab (a bc b ca c ab)

Phép chứng minh được hoàn tất .

3.Các kĩ thuật sử dụng Bất Đẳng Thức Cô si :

1.Kĩ thuật chọn điểm rơi AM-GM:

Trước hết, xin nhắc lại rằng : Điều quan trọng khi giải toán Bất Đẳng Thức là các đánh giá

trung gian phải đảm bảo được dấu đẳng thức xảy ra . Chính vì thế, việc đoán được dấu

đẳng thức xảy ra giúp ta định hướng tốt hơn lời giải . Ta tạm gọi đó là việc dự đoán điểm

rơi .

Trong các bất đẳng thức dấu “ ” thường xảy ra ở các trường hợp sau:

Các biến có giá trị bằng nhau. Khi đó ta gọi bài toán có cực trị đạt được tại tâm

Khi các biến có giá trị tại biên ( 1 biến bằng 0). Khi đó ta gọi bài toán có cực trị đạt được tại

biên

Ngoài ra , củng có một số trường hợp ngoại lệ là 3 biến lệch nhau hoàn toàn . Không có một

“thuật toán” nào có thể giúp chúng ta dự đoán được dấu bằng bằng tay cả . Nếu dùng máy

tính thì chúng ta có thuật toán Fermat-Lagrange để làm điều này . Nhưng chúng ta củng có

thể có một vài cách tư duy để dự đoán được dấu bằng . Trường hợp tầm thường nhất đó là

dấu đẳng thức xảy ra tại tâm 3 biến bằng nhau . Điều này thường xảy ra đối với các bài toán

đối xứng 3 biến ( vai trò a,b,c như nhau ) . Trường hợp, hay gặp thứ 2 là có một biến bằng 0.

Trong trường hợp này, gần như BĐT AM-GM không làm gì được và nó trở nên không đủ

sức công phá các bài dạng này . Ta sẽ nói ở sau về dạng bài này . Trong một số bài toán có

điều kiện kiểu như 3 biến a,b,c thuộc một đoạn đóng nào đó kiểu a;b thì rất có thể đẳng

thức sẽ xảy ra tại 2 điểm đầu và cuối , và biến còn lại chúng ta có thể hoàn toàn tìm ra được

bằng cách thử trực tiếp . Hoặc giả như, với các BĐT không đối xứng 3 biến thì hãy cố tìm 2

biến mà nó đối xứng nhau trong 3 biến đó và 2 biến đối xứng này sẽ bằng nhau,và hãy gán

cho nó một giá trị . Sau đó ta chỉ tìm cực trị của biểu thức 1 biến .Điều này khá đơn giản bởi

sự hỗ trợ của đạo hàm.

Bây giờ, ta sẽ tìm hiểu kĩ thuật chọn điểm rơi với BĐT AM-GM :

Ví dụ 1: Cho 2a . Tìm giá trị nhỏ nhất của biểu thức: 2

1S a

a

Lời giải

1 1 7 1 7 2 7 2 7 2 2 7 92

2 2 28 8 8 8 8 8 4 4 48 8 2

a a a a a .S a .

a .a a a

9

4minS

Lời giải này là lời giải sai .Ta phân tích kỹ hơn : Mặc dù chọn điểm rơi 2a và 9

4minS là

đáp số đúng nhưng cách giải trên đã mắc sai lầm trong việc đánh giá mẫu số: Nếu 2a thì

2 2 2

48 8 2a . là đánh giá sai.

Để thực hiện lời giải đúng ta cần phải kết hợp với kỹ thuật tách nghịch đảo, phải biến đổi S

sao cho sau khi sử dụng BĐT Côsi sẽ khử hết biến số a ở mẫu số.

Lời giải đúng:

1 1 6 1 6 3 6 3 6 2 933

2 2 28 8 8 8 8 8 4 8 4 8 4

Côsia a a a a a a .S a . .

a a a

Với 2a thì 9

4minS

Ví dụ 2: Cho0

3

2

a, b, c

a b c

. Tìm GTNN của 1 1 12 2 22 2 2

S a b cb c a

Lời giải

Do S là một biểu thức đối xứng với a, b, c nên dự đoán MinS đạt tại 12

a b c

Bây giờ, ta thay điểm rơi vào ta có phép tính :1

44 .Như vậy ta sẽ tách 4 để sao cho có

1

4

.Vậy ta đi đến lời giải sau :

1 1 1 1 1 12 2 22 2 2 2 2 216 16 16 16 16 16

16 16 16

S a ..... b ..... c .....b b c c a a

1 1 1 1 1 12 2 217 17 172 2 2 2 2 217 17 1716 16 16 16 16 16

16 16 16

a . ..... b . ..... c . .....b b c c a a

2 2 217 17 17 17 17 1717 17 17 17

16 32 16 32 16 32 8 16 8 16 8 1616 16 16 16 16 16

a b c a b c

b c a b c a

3 17317 17 17 1717 3 3 17

8 16 8 16 8 16 8 5 5 5 516 16 16 16 172 2 2 2

a b c a. . .

b c a a b c . a b c

3 17 3 17

15 22 2 2172

3

a b c.

.

Dấu “ = ” xảy ra khi 1

2a b c Min S =

3 17

2

Tiếp theo ta xét một VD kinh điển sau, là bài toán dự bị đề thi quốc tế năm 1998 .

Ví dụ 3: Chứng minh rằng với mọi số dương x, y,z thõa mãn 1xyz thì :

33 3 3

1 1 1 1 1 1 4

yx zA

( y)( z) ( z)( x) ( x)( y)

Lời giải

Ở bài toán này, không có sự xuất hiện của những căn thức . Nhưng lại xuất hiện một biểu

thức mẫu khá là khó chịu . Ta thử đánh giá mất mẫu xem thế nào .

Theo lối tư duy này ta xét riêng phân thức : 3

1 1

x

( y)( z) ( các phân thức khác tương tự )

Để có thể mất đi mẫu 1 1( y)( z) thì ta sẽ tìm cách nhân phân thức với biểu thức

1 1( y)( z)

Như vậy nghĩa là ta tìm một BĐT nào đó liên quan đến phép nhân,hiển nhiên đơn giản nhất

đó chính là Bất Đẳng Thức AM-GM .Vấn đề tiếp theo cần xác định là áp dụng BĐT AM-GM

với bộ bao nhiêu số ?

Để ý điều kiện đề bài là tích 3 số (ở bậc 1),trong khi đó,số mũ của tử số là mũ 3,vì vậy,trong tư tưởng

của ta,việc đánh giá phải làm mất được mẫu và đưa tử về bậc 1 (hoặc lớn hơn)-Tuy nhiên,chẳng dại

gì mà ta lại đánh giá để đưa về bậc lớn,vì càng lớn thì việc khử "phần thêm" sẽ càng khó,Vậy,ta sẽ áp

dụng cho 3 số(Sau này,trong hầu hết các bài toán đánh giá mẫu,thường thì bậc của tử là bậc bao

nhiêu thì sẽ đánh giá cho từng ấy số hạng )

Từ những phân tích trên, thì hướng đánh giá của chúng ta sẽ là như sau :

Áp dụng Bất đẳng thức AM-GM cho 3 số dương, ta có :

3 1 1 3

1 1 8 8 4

yx z x

( y)( z)

Xây dưng các BĐT tương tự rồi cộng lại, ta có :

3

2 4

x y zA

Để ý rằng : 33 3x y z xyz

Vậy ta có

3

4A

Dấu đẳng thức xảy ra tại 1x y z

4. Kĩ thuật ghép đối xứng

Với một số Bài toán chứng minh BĐT mà cả Vế trái lẫn vế phải đều là những biểu thức phức

tạp việc chứng minh trực tiếp khá khó khăn. Thì ta có thể sử dụng kĩ thuật ghép đối xứng để

đơn giản hóa công việc chứng minh. Một cách đơn giản, ta có thể chia thành 2 dạng như

sau:

Dạng 1: Chứng minh X Y Z A B C

Công viêc chúng ta là tìm cách chứng minh: 2X Y A . Nhờ tính đối xứng ta thiết lập

thêm được 2 bất đẳng thức tương tự rồi cộng lại , khi đó ta có được điều phải chứng minh.

Dạng 2: XYZ ABC

Ta thử tìm cách chứng minh: 2XY A .Tương tự ta có thêm 2 bất đẳng thức tương tự. Sau

đó nhân từng vế bất đẳng thức cùng chiều, ta có điều phải chứng minh.

Ta hãy đón xem các ví dụ sau

Mở đầu: Chứng minh rằng với mọi 0a,b,c ,thì ta có: abc (a b c)(b c a)(c a b)

Lời giải

Đây là 1 BĐT có rất nhiều ứng dụng đã được đề cập đến trong SGK toán lớp 10.

Trước hết nếu như 0(a b c)(b c a)(c a b) thì Bài toán là hiển nhiên. Bây giờ ta

xét trường hợp ngược lại, tức là 0(a b c)(b c a)(c a b)

Vận dụng kĩ thuật, ghép đối xứng ta chứng minh:

2b (a b c)(b c a) .

May mắn thay là điều này hiện nhiên đúng theo BĐT quen thuộc sau:2

4

(x y)xy

Như vậy ta có phép chứng minh hoàn tất và ta có đpcm.

Ví dụ 1: Cho các số thực không âm a, b,c thõa mãn 0ab bc ca . Chứng minh rằng:

2 2 21 1 13

a b c

b c c a a b

Lời giải

Khi ra cho học sinh Bài toán này, chúng tôi lập tức nhận được đề xuất ý tưởng giải là áp

dụng BĐT Am-GM kiểu: 21 2a a . Để quy về chứng minh bất đẳng thức sau:

23

a

b c

Và “đâm đầu” chứng minh Bất đẳng thức trông có vẻ rất đúng và….đẹp này. Nhưng nếu

như cho 0a b,c ta thấy ngay bất đẳng thức này……không đúng.

Như vậy, các bạn học sinh đã lặp lại lối mòn muôn thuở là “ngược dấu”.

Tôi đã hướng dẫn các em làm “chặt” đánh giá của mình hơn bằng cách áp dụng trực tiếp.

Điều này làm giảm rủi ro hơn ( vì nó không làm quá lỏng Bài toán ).

Như vậy, theo AM-GM , ta có:

2 2 22 2 2

31 1 11 1 1

3( a )( b )( c )a b c

b c c a a b (a b)(b c)(c a)

Bây giờ, phép chứng minh hoàn tất nếu ta chỉ ra được:

2 2 21 1 1( a )( b )( c ) (a b)(b c)(c a)

Sử dụng ghép đối xứng, ta thử kiểm tra BĐT sau đúng không ?

2 2 21 1( a )( b ) (a b)

Nhân “tung tóe” ra ta được bất đẳng thức trên tương đương với: 21 0(ab )

Như vậy ta có đpcm

Ví dụ 2: Cho a, b,c là các số thực dương. Chứng minh rằng:

2 2 2 2 2 2

2 2 28

(a b)(b c)(c a) (a b )(b c )(c a )

abc (a bc)(b ca)(c ab)

Lời giải

Nhân chéo lên ,thì bất đẳng thức cần chứng minh được viết lại thành:

2 2 2 2 2 2 2 2 28(a bc)(b ca)(c ab)(a b)(b c)(c a) abc(a b )(b c )(c a )

Ta thử chứng minh bất đẳng thức sau đúng:

2 2 2 2 22(c ab)(a b) ab(b c )(c a )

Dễ thấy vế phải có cấu hình khá rỏ ràng là 2 xy ,nên gợi ý ta sử dụng AM-GM.

Để làm được điều này, ta tách VP thành 2 số hàng.

Để ý: 2 2 2 2 2 2 2 2 22(c ab)(a b) a(b c ) b(c a ) ab(b c )(c a )

Vậy ta có điều phải.

Ví dụ 3: Cho a, b,c là các số dương. Chứng minh rằng:

2 2 2a bc b ca c ab abc a b b c c a

Lời giải

Đối với những Bài toán dạng này thì kĩ thuật ghép đối xứng là kĩ thuật mà ta nên nghĩ tới.

Nghĩa là ta tách bên vế trái thành từng cặp rồi tìm đánh giá sao cho hiệu quả.

Quan sát, ta thấy rằng:

2 2 2 2 3 3 2

2 2 2 2 2

2 2 2

(a bc)(b ca) a b c(a b ) abc

a b c(a b)(a ab b ) abc

a b c(a b)ab abc ab(a c)(b c)

Tương tự, thiết lập thêm 2 BĐT nữa, ta có ngay điều phải chứng minh.

Ví dụ 4: Cho a, b,c là các số thực dương thõa mãn 3a b c .Chứng minh rằng:

3a b b c c a

c ab a bc b ac

Lời giải

Áp dụng Bất Đẳng Thức AM-GM , ta cần phải chứng minh:

(a b)(b c)(c a) (c ab)(b ca)(c ab)

Bây giờ ta sẽ chứng minh:

(a b)(b c) (c ab)(a bc)

Khai triển ra tương đương với: 2 2

22 2 2

3

32 2 3 2 3

4

c a abc

( b)(c a) ac abc (c a) ac(b ) ( b) (b )

Việc kiểm tra hàm 1 biến còn lại , xin dành cho bạn đọc xem như luyện tập

Ví dụ 153: Cho các số thực dương x, y,z thỏa mãn : 2 2 25 9 2x y z xy yz zx .

Tìm giá trị nhỏ nhất :

32 2

1x

y z x y z

Nhận xét và lời giải:

Bây giờ ta xét sang một kĩ thuật đơn giản nhưng có khá nhiều ứng dụng hay

Sử dụng tính chất của hàm số bậc nhất và hàm số bậc hai

Những công cụ tưởng chừng như đơn giản nhất đôi khi lại mang đến những sức mạnh cho

chúng ta . Trong tiết này, chúng ta sẽ theo dõi ứng dụng của hàm số bậc nhất và hàm số bậc

2 trong chứng minh các bài toán bất đẳng thức

** Hàm số bậc nhất: là hàm số có dạng: y f x ax b . Ta có:

- Nếu 0a thì hàm số đồng biến trên

- Nếu 0a thì hàm số nghịch biến trên

Từ hai tính chất trên ta có:

Hàm số y f x ax b có TXĐ: D ; thì:

+ Xét 0a hàm số đồng biến trên

y

y

min f;

max f

+ Xét 0a hàm số nghịch biến trên

y

y

min f;

max f

Tóm lại, ymax max f ; f và ymin min f ; f

Tính biến thiên của hàm số bậc nhất được ứng dụng vào chứng minh BĐT thông qua 2

mệnh đề sau:

BĐT: f x k đúng với mọi x D f x min k

BĐT: f x k đúng với mọi x D f x max k

** Hàm số bậc 2: là hàm số có dạng: 2 0y f x ax bx c a

Xét hàm số 2 0y f x ax bx c a có TXĐ: D ; thì:

Nếu

0

2

y

y

max max f ; f

a bmin min f ; f ; f

a

Nếu

0

2

y

y

min min f ; f

a bmax max f ; f ; f

a

Kết quả trên có thể chứng minh bằng bảng biến thiên xin dành cho các bạn!

Ứng dụng của hàm số bậc nhất và hàm số bậc hai trong nhiều trường hợp là tư duy đơn

giản và thuần túy:

Bài 1: Cho 0 1x, y,z ; . Chứng minh rằng:

1x y z xy yz zx

Lời giải

Nếu phân tích hằng đẳng thức là điều quan trọng thì trong trường hợp này nó là một trong

những hướng đi được nghĩ đến:

Sử dụng phân tích: 1 1 1 1x y z x y z xy yz zx xyz

Ta thấy ngay đpcm!

Tuy nhiên, thử ứng dụng hướng đi dùng t/c hàm số bậc nhất trong trường hợp này:

Xem x là ẩn, ta có: 1 1f x y z x y z yz với 0 1x, y,z ;

Ta có: 0 1 1 1 0f y z yz y z luôn đúng vì 0 1y,z ;

Và 1 0f yz , Theo kết quả trên ta có ngay 0f x với mọi 0 1x, y,z ;

Từ đó ta có đpcm!

Bài 2: Cho a,b,c là các số thực thuộc đoạn 1 2; và 0a b c . Tìm GTLN của:

2 2 2P a b c

Lời giải

Ta có: 22 2 2 22P a b a b a ab b với 1 2a,b ; và 2 1a b ;

Xét hàm số 2 2f a a ab b với 1 2a ;

Suy ra 21 1f a max f b b với 1 2b ;

hoặc 22 2 4 3 6f a max f b b P max (vì 2 1 1a b ; b )

Xét 2 hàm số 2 1g b b b với 1 2b ;

Có:

1 33 6

2 3

g b max gg b max P max

g b max g

Vậy 6P max tại 2 1 1a,b,c ; ; và các hoán vị của nó!

Bài 3: Cho a,b,c là các số không âm có tổng bằng 1. Chứng minh rằng:

70 2

27ab bc ca abc

Bài toán ta đã gặp không ít lần trong cuốn sách này, sau đây là cách giải theo hướng đi này:

Lời giải

Xét hàm số 7

227

f ab ab bc ca abc = 7

1 2 127

c ab c c

là hàm số bậc nhất có: 0 1ab ; và 0 1c ;

Ta có: 2

7 1 10 1 0

27 2 108f c c c

với mọi 0 1c ;

Và: 2

7 1 1 11 1 2 1 0

27 2 3 6f c c c c c

với mọi 0 1c ;

Theo kết quả trên ta có ngay 0f ab với mọi a,b,c. Từ đó ta có đpcm!

Bài 4: Cho a,b,c là các số thực dương thỏa mãn 4ab bc ca abc .

Chứng minh rằng: a b c ab bc ca

Bài toán từng được sử dụng nguyên lí Dirichlet. Sau đây là lời giải trên tạp chí THTT:

Lời giải

Rút 4 ab

ca b ab

. Ta cần chứng minh:

1 44

ab aba b

a b ab

Ta có: BĐT 1 4 4a b a b ab ab ab a b ab

4 1 4 0a b a b ab ab ab

Đặt 2

4

Sa b S,ab P P . Ta có:

2

0 4 44

Sc P P min ;

Và BĐT cần c/m 4 1 4 0S S P P P

Dễ thấy đây là hàm số bậc 2 ẩn P, có hệ số cao nhất âm và 2

0 44

SD ;min ;

Ta chỉ cần chứng minh 0 0f và 2

4 04

Sf min ;

.

Ta có: 2

0 2 0f S đpcm!

Với chứng minh 2

4 04

Sf min ;

:

+ Nếu 2

4 4 4 4 4 04

SS f min ; f S S

đpcm!

+ Nếu

222 2 16 24 4 0

4 4 4

S SS SS f min ; f

đpcm!

Vậy ta có đpcm. Đẳng thức xảy ra khi và chỉ khi 1a b c

Bài 5: Cho a,b,c không âm và 1a b c k k . Chứng minh:

33 3 3 6

4

ka b c abc

Đầu tiên biến đổi VT về dạng PT bậc nhất hoặc bậc 2. Một bài toán tổng quát rất đẹp!

Lời giải

Ta có:

3 3 3 2 2 26 9a b c abc a b c a b c ab bc ca abc

2

3 9a b c a b c ab bc ca abc

3 3 33 9 3 3 3 3 3 3 1k k ab bc ca abc c k ab kc a b k c k ab kc c k

Xét hàm số 33 3 3 1f ab c k ab kc c k

với 0c ;k và 2 2

210

4 4 4

k kab a b ab ;

Ta có: 2

3 3 31 3 30 3 1 3

2 4 4f kc c k k c k k k k

Lại có 3

3 331

4 4

kk k k k k luôn đúng theo giả thiết!

2 2

3 2 2 33 13 3 3 1 3 3

4 4 4 4

k kf c k kc c k kc k k c k

33 13 1

4 4kc c k k

Điều ta cần bây giờ là 3 1

1 04 3

c k c k . Điều này hoàn toàn có thể:

Giả sử 33

kc max a; b; c c a b c k c

Suy ra

2

3 3 33 1 1 13 1 3 1

4 4 4 4 4

kf kc c k k kc k k k

Vậy từ nhận xét đầu bài viết ta có ngay đpcm!

Dấu “=” có 1 1

1 02 2

k ; a; , b, c ; ;k

và các hoán vị của nó

Tiếp theo ta khảo sát một lớp bài toán là ứng dụng của tam thức bậc 2 để chứng minh các

bài toán về Bất đẳng thức , cực trị :

Bài 1. Chứng minh rằng với x,y,z là các số thực có tổng bằng 1 ta có

2

3 4 5 44x y z xy yz zx .

Lời giải

Thay 1z x y bất đẳng thức trở thành:

2

2 2

3 4 5 5 5 44 44 1

48 16 3 4 45 54 25 0

x y x y xy x y x y

x x y y y

.

Vế trái là tam thức bậc hai của x với hệ số của 2x dương và có

2 2264 3 4 48 45 54 25 176 3 1 0x' y y y y .

Vậy bất đẳng thức được chứng minh. Đẳng thức xảy ra khi và chỉ khi 1 1 1

2 3 6x , y ,z .

Cách 2: Bất đẳng thức đã cho tương đương với:

2 2 29 16 25 20 4 14x y z xy yz zx .

Sử dụng bất đẳng thức AM – GM ta có

2 2 2 2

2 2 2 2

2 2 2 2

5 54 9 2 4 9 20

3 37 7

4 36 2 4 36 1412 12

4 2 4 4

x y . x . y xy

x z . x . z xz

y z y . z yz

.

Cộng theo vế ba bất đẳng thức trên ta có đpcm. Như vậy không cần giả thiết bài toán ba số

có tổng bằng 1(Xem thêm chủ đề kỹ thuật tham số hoá – Chương 2).

Bài 2. Chứng minh rằng với mọi số thực a và b ta luôn có

2 2 2 23 1 1 2 1a a b b ab a b .

Lời giải

Viết lại bất đẳng thức dưới dạng: 2 2 2 23 3 3 5 3 3 3 1 0a a b a a b a a .

Vế trái là tam thức bậc hai của b có 2 3 3 0a a , a và;

2 22 2 2 23 5 3 4 3 3 3 3 1 3 1 0b a a a a a a a a , a .

Do đó vế trái luôn không âm. Bài toán được chứng minh.

Đẳng thức xảy ra khi và chỉ khi

2

2

2

3 1 03 5

3 5 32

2 3 3

a a

a ba ab

a a

.

Bài tập tương tự

Chứng minh rằng với mọi số thực a,b,c,d ta có

2 2 2 2 2 2 2 23 2(a ab b )(c cd d ) (c a abcd b d ) .

Ta cùng xét một số bài toán cùng dạng sau đây

Kỷ thuật hệ số không xác định (UCT)

Đây là một kĩ thuật khá là mạnh trong chứng minh bất đẳng thức . Ý tưởng của phương pháp này

củng chính là việc đánh giá đại diện theo từng biến . Nói như vậy thì nhiều bạn sẽ bảo là đã có kĩ

thuật tiếp tuyến rồi chúng ta không cần tham khảo thêm nữa . Nhưng xin thưa rằng : Đánh giá tiếp

tuyến đôi lúc không mang lại kết quả vì nó không luôn giữ dấu lớn hơn hoặc luôn giữ dấu bé hơn , nó

sẽ biến thiên theo từng khoảng đang xét . Điều này khá là khó xử lý cho chúng ta . Xin cảm ơn anh

Nguyễn Thúc Vũ Hoàng ( hàng xóm của tác giả ở quê ) đã cung cấp tài liệu và cho phép chúng tôi sử

dụng tài liệu của mình trong quá trình biên soạn chuyên đề này . Xin phép trích dẫn phương pháp

này cùng với một số bài tập minh họa của nhóm biên soạn gửi đến bạn đọc . Lưu ý khi đọc chương

này, các bạn sẽ cảm thấy khó hiểu nhưng không sao, hãy đọc kĩ lại chúng ta sẽ thấy được nét đẹp của

nó .

Ví dụ 1: Cho a, b,c là các số thực dương thỏa mãn 3a b c . Chứng minh rằng

2 2

2 2 2

1 1 15

3

22(a b c )

a b c

Lời giải

Ta sử dụng bất đẳng thức sau đây

2

2

1 2 7 2

3 3 3

a a

a

Thật vậy bất đẳng thức trên tương đương với

2 2

2

1 2 6 30

3

(a ) ( a a )

a

Hiển nhiên đúng với a là số thực dương.

Sử dụng các bất đẳng thức tương tự với b và c. Ta có điều phải chứng minh.

Đẳng thức xảy ra khi 1a b c .

Chắc chắn ngay khi đọc lời giải cho bài toán “ đơn giản” này bạn có phần lúng túng và không

hiểu tại sao lại có thể tìm ra bất đẳng thức phụ một cách “khó hiểu” như vậy. Phải chăng là

dự đoán một cách “vô hướng”. Hoặc cũng có người sẽ nghĩ bài toán trên được tạo ra từ chính

bất đẳng thức phụ đó. Câu trả lời là hoàn toàn không phải. Tất cả đều đi theo 1 qui luật của

nó. Ở các phần tiếp theo chúng tôi sẽ phân tích về một kỹ thuật phân tích giúp tìm ra các bất

đẳng thức phụ và mở rộng vấn đề này theo chiều hướng khá mới mẻ. Kỹ thuật này có tên là

U.C.T, là viết tắt của 3 chữ cái đầu của cụm từ tiếng Anh Undefined Coefficient Technique.

Hay còn gọi là Kỹ Thuật Hệ số bất định. Đây là một kỹ thuật cơ bản và là nền tảng quan trọng

trên con đường tìm kiếm lời giải cho những bất đẳng thức khó.

Chúng ta sẽ khởi đầu kỹ thuật này bằng việc đưa ra cách giải thích cho việc tìm ra bất đẳng

thức phụ trên và nó cũng chính là cách giải thích cho các bài toán sau này của chúng ta.

Bài toán trên các biến trong cả 2 vế và điều kiện đều không ràng buộc nhau điều này khiến ta

nghĩ ngay sẽ tách theo từng biến để chứng minh được đơn giản hơn nếu có thể. Nhưng rõ

ràng ta chỉ từng đó thôi là không đủ. Nếu ta chứng minh bất đẳng thức sau

22

2 2

1 1 2 31 2 50

3 3 3

(a )(a )( a )a

a a

Rõ ràng không hoàn toàn đúng với a thực dương.

Đừng bỏ cuộc tại đây bởi vì ở cách trên ta chưa sử dụng điều kiện 3a b c .

Như vậy ta sẽ không đi theo đường lối suy nghĩ đơn giản ban đầu nữa mà sẽ đi tìm hệ số để

bất đẳng thức sau là đúng

2

2

1 2 5

3 3

ama n

a (1)

Trong đó m và n là các hệ số chưa xác định.

Tương tự với biến b và c. Cộng vế theo vế ta có

2 2 2

2 2 2

1 1 1 2 2 2 5 53 3

3 3 3

a b cm(a b c) n (m n)

a b c

Như vậy ở đây 2 hệ số m và n phải thỏa mãn điều kiện 0m n n m . Thế vào (1) dẫn

đến

2

2

1 2 51

3 3

am(a )

a (2)

Đến đây ta chỉ cần xác định hệ số duy nhất là m để bất đẳng thức (2) là đúng.

Chú ý ở bài toán này điểm cực trị đạt được tại 1a b c nên ta cần xác định m sao cho

22

2 2

1 2 31 2 51 1 0

3 3 3

(a )( a )am(a ) (a ) m

a a

Khi cho 1a thì ta có 2

2

1 2 3 2

3 3

(a )( a )

a

từ đó ta dự đoán rằng

2

3m để tạo thành đại

lượng bình phương 21(a ) trong biểu thức. Từ đó ta sẽ chứng minh bất đẳng thức phụ

2

2

1 2 7 2

3 3 3

a a

a

Quá trình đi tìm bất đẳng thức phụ đã được phân tích cụ thể ở trên. Tuy nhiên đó không phải

là cách duy nhất để ta tìm ra hệ số. Ta cũng có thể sử dụng tính chất của đường tiếp tuyến tại

một điểm của đồ thị hay sử dụng đạo hàm. Nhưng có lẽ cách dự đoán trên là hữu hiệu và đơn

giản về mặt trực quan cũng như thực hiện. Tuy nhiên tất cả cũng chỉ là sự dự đoán. Nó không

đảm bảo rằng sau khi tìm ra bất đẳng thức phụ rồi thì bài toán sẽ được giải quyết. Một số

dạng toán như vậy sẽ được đề cập trong các phần tiếp theo của chuyên đề này. Ở phần 1 này

chúng ta sẽ chứng minh một số bất đẳng thức cơ bản đề hình thành trong đầu kỹ thuật qua

đó thành thục trong việc phân tích. Ta tiếp tục đến với bài toán sau

Ví dụ 2 : Cho a,b, c,d là các số thực dương thỏa mãn 4a b c d . Chứng minh rằng

2 2 2 2

1 1 1 12

1 1 1 1a b c d

Lời giải

Ta sẽ xác định hệ số m để bất đẳng thức sau là đúng

2 2 2

1 12 11 1 1 1 0

1 1 1

(a )(a ) am(a ) m(a ) (a ) m

a a a

Khi 1a ta sẽ có 2

11 1

1

am

a

. Ta dự đoán bất đẳng thức sau đúng và thật vậy

2

2 2

122 0

1 1

a(a )a

a a

Tương tự với các biến còn lại. Cộng vế theo vế ta có điều phải chứng minh.

Đẳng thức xảy ra khi và chỉ khi 1a b c d .

Bình luận : Ta có thể sử dụng kỹ thuật “Côsi ngược dấu” để tìm ra bất đẳng thức phụ trên

2 2

2 2

11 1 1

1 1 2 2

a a a

a a a

Ví dụ 3: Cho a, b,c là các số thực dương thỏa mãn 3a b c . Chứng minh rằng

2 2 2

1 1 11

a b c b c a c a b

Lời giải

Ở đây ta cần tìm m để bất đẳng thức dưới là đúng

2 2 2

11 1 11 1

3 3 3 3

a(a )m(a ) m(a )

a b c a a (a a )

Tương tự như trên ta tìm dự đoán rằng với 1

9m thì bất đẳng thức phụ đúng. Thật vậy

2 2

2 2 2

1 3 11 40 0

3 9 9 3 3 3 3

(a ) ( a) (a ) (b c)a

a a (a a ) (a a )

Ví dụ 4: Cho a,b, c,d là các số thực không âm thỏa 2 2 2 2 4a b c d . Chứng minh rằng

3 3 3 3 32 2 2

2(a b c d ) ab ac ad bc bd dc

Lời giải

Theo bài ra a,b, c,d là các số thực dương thỏa mãn

2 2 2 2

2

4

2 2

2 2

a b c d

(a b c d) ( ab ac ad bc bd cd)

(a b c d) ( ab ac ad bc bd cd)

Bất đẳng thức cần chứng minh tương đương với

3 3 3 3 32 2

2(a b c d ) (a b c d)

Ta cần xác định hệ số m để bất đẳng thức sau đúng

23 2 1 13 1

2 1 12 2

( a ) (a )aa m(a ) m(a )

Dễ dàng dự đoán 9

2m . Ta sẽ chứng minh điều đó, thật vậy

3 29 13 12 2 1 2 0

2 2

(a )aa (a ) (a )

Điều này hiển nhiên đúng. Đẳng thức xảy ra khi và chỉ khi 1a b c d .

Nhận xét. Bài toán này với hình thức khá “cồng kềnh” vì chứa căn thức. Tuy nhiên nếu nhận

ra điểm mấu chốt của bài toán ta dễ dàng đưa về đơn lượng theo biến để giải quyết. Bài toán

trên còn có thể giải quyết theo cách khác bằng cách chứng minh trực tiếp với 4 biến. Nhưng

dù sao việc giải quyết theo từng biến riêng biệt vẫn dễ dàng hơn rất nhiều.

ĐẶT MUA SÁCH

Link đăng ký: http://goo.gl/forms/5SbEpf57U9

Mua trực tiếp liên hệ Nguyễn Văn Quốc Tuấn số điện thoại: 0989631669

Facebook: https://www.facebook.com/chicanemhanhphucS2g

Đặt mua sách Bí quyết tiếp cận hiệu quả Kỳ thi THPT quốc gia Bất đẳng thức- Giá trị lớn nhất

nhỏ nhất.

Nếu mua qua đây sẽ được tác giả ký tặng!!!

------GIÁ SÁCH---

Lưu ý: Mua nhóm trên 5 cuốn được miễn phí cước Chuyển phát nhanh.

Bí quyết tiếp cận hiệu quả Kỳ thi THPT quốc gia Bất đẳng thức- Giá trị lớn nhất nhỏ nhất. Giá

bìa 234.000đ

----THANH TOÁN----

TỔNG TIỀN THANH TOÁN = 200K +40K phí Chuyển phát nhanh, Bạn nào lựa chọn thanh

toán COD Qua bưu điện thì mất thêm 15K tiền thu hộ cho nhân viên bưu điện và cần thanh

toán trước 20K bằng cách gửi MÃ THẺ CÀO +SERI thẻ cào điện thoại vào số 0989631669.

---CÁCH THỨC THANH TOÁN ---

Hình thức 1: CHUYỂN KHOẢN

Thông tin tài khoản của thầy:

Các bạn chuyển vào tài khoản sau:

+Chủ thẻ: Nguyễn Văn Quốc Tuấn

+Ngân hàng TMCP Công Thương Việt Nam (Viettinbank) - Loại tài khoản: A - TK ATM

Số TK: 711AB2793863

HÌNH THỨC 2: THANH TOÁN BẰNG THẺ CÀO:

Sau khi đặt sách các bạn Gửi Mã thẻ cào + Số Seri (áp dụng với tất cả các loại thẻ của nhà

mạng) vào số điện thoại 0989631669. Lưu ý: Thanh toán bằng thẻ cào các em thanh toán 130%

tổng giá thanh toán gồm 130%*(200 ngàn +40 ngàn phí ship)

Lưu ý: Mua nhóm trên 5 cuốn được miễn phí vẫn chuyển